Causalidad y teoría cuántica de campos [duplicado]

Tengo un problema con la prueba de causalidad en Peskin & Schroeder, An Introduction to QFT, página 28. Para evitar confusiones, utilizo la notación de tres vectores, reescribiendo la ecuación. (2.53) para y = 0 como sigue:

[ ϕ ( X , t ) , ϕ ( 0 , 0 ) ] = d 3 pag ( 2 π ) 3 1 2 pag 2 + metro 2 ( mi i pag . X i t pag 2 + metro 2 mi i pag . X + i t pag 2 + metro 2 )

El libro continúa sobre cómo el integrando siendo invariante de Lorentz hace que esta integral sea cero para la x fuera del cono de luz. Pero yo (que no soy un experto en relatividad especial) quiero verlo con más rigor:

después de cambiar las variables pag pag en el primer término, la ecuación se simplifica a:

[ ϕ ( X , t ) , ϕ ( 0 , 0 ) ] = d 3 pag ( 2 π ) 3 2 i 2 pag 2 + metro 2 mi i pag . X pecado ( t pag 2 + metro 2 )

usando coordenadas esféricas:

[ ϕ ( X , t ) , ϕ ( 0 , 0 ) ] = d pag d ϕ d θ pag 2 pecado θ ( 2 π ) 3 i pag 2 + metro 2 mi i pag X porque θ pecado ( t pag 2 + metro 2 ) [ ϕ ( X , t ) , ϕ ( 0 , 0 ) ] = 0 d pag pag ( 2 π ) 2 2 i X pag 2 + metro 2 pecado ( pag X ) pecado ( t pag 2 + metro 2 )

de nuevo después de otro cambio de variables tu = pag 2 + metro 2 ,

[ ϕ ( X , t ) , ϕ ( 0 , 0 ) ] = 2 i X metro d tu ( 2 π ) 2 pecado ( X tu 2 metro 2 ) pecado ( t tu )

No puedo ver cómo esta integral debería ser cero para X > t !!! ¿Puede alguien por favor explicarme esto?

Respuestas (2)

Abordaré su punto sobre por qué la integral es invariante de Lorentz, a partir de los comentarios a la respuesta de cduston, creo que este es su punto de conflicto:

Puedes ver la relación entre una forma manifiestamente invariante de Lorentz como esta

d 4 pag mi i pag X ( p 2 m 2 )       ( 1 )
and the not-so-obviously Lorentz invariant form
d 3 p 1 E p e i p x       ( 2 )
by using the identity
1 ( p 2 m 2 ) = 1 2 E pag { 1 ( mi pag + pag 0 ) 1 ( mi pag pag 0 ) }
Aquí mi pag = pag 2 + metro 2 es el componente de tiempo en el caparazón del cuatro vector de impulso, y pag 0 es el componente de tiempo "genérico", no necesariamente en el shell.

Si sustituye esto en (1) y hace lo pag 0 integral usando el contorno apropiado, obtendrás (2).

Lo que realmente está pasando se explica en la discusión cerca de la ecuación (2.40), estás haciendo una integral de 4 impulsos, pero solo restringiéndola a la capa de masa usando una función delta. La restricción a un caparazón de masa es una operación invariante de Lorentz, por lo que está manteniendo la invariancia de Lorentz en todo momento (¡aunque con la integral de tres momentos no lo parece!).

En el texto dice que los dos términos se desvanecen bajo ( X y ) ( X y ) . En otras palabras, hay una transformación de Lorentz que toma ( X y ) ( X y ) en el segundo término cuando la separación es espacial ( ( X y ) 2 < 0 usando el signo equivocado...). Haz eso, y el conmutador desaparece.

I agree with your comment in the text says when the separation is space like one can do such a transformation and get zero. Provided that the term is Lorentz invariant. I cannot see how this term is Lorentz invariant. I can see how d 3 p f ( p ) / p 2 + m 2 is Lorentz invariant but not a function like d 3 p f ( p , x , t ) / pag 2 + metro 2 . ¿Podrías explicarme por qué? d 3 pag mi i pag . X i t pag 2 + metro 2 / pag 2 + metro 2 ¿Lorentz es invariante?
La cosa en el exponente es solo el producto interno invariante de Lorentz entre el 4-momentum pag m = ( pag , pag 2 + metro 2 ) y el de 4 posiciones X m = ( X , t ) (hasta cualquier índice y convención de signos que estén usando).
El cartel anterior es correcto, pero PS también menciona esto unas páginas antes en su texto. Ahora mismo no lo tengo, pero échale un vistazo.
@Blackie Eq.(2.40) en la página 23 muestra que es invariante de Lorentz.